How Can We Show the Bound for $\delta_{h,-,2} f(x)$?

  • MHB
  • Thread starter evinda
  • Start date
  • Tags
    Bound
In summary, the formula $\delta_{h,-,2} f(x) :=- \left( \delta_{h,-}+\frac{h}{2} \delta_{h,-}^2 \right) f(x)=\frac{1}{2h} \left( -f(x-2h)+4f(x-h)-3f(x)\right)$ provides an approximation of $f'(x)$ with an error term of $h^2 ||f'''||_{\infty}$. Even though the formula may seem to be calculating $-\delta_{h,-,2} f(x)$ instead of $\delta_{h,-,2} f(x)$, the error term ensures that it is still a valid
  • #1
evinda
Gold Member
MHB
3,836
0
Hello! (Wave)

We define $\delta_{h,-,2} f(x) :=- \left( \delta_{h,-}+\frac{h}{2} \delta_{h,-}^2 \right) f(x)=\frac{1}{2h} \left( -f(x-2h)+4f(x-h)-3f(x)\right)$.

Let $f \in C^3[a,b]$. Then:

$$| \delta_{h,-,2} f(x)- f'(x)|\leq h^2 ||f'''||_{\infty}$$

I have tried the following:

$$f(x-h)=f(x)-h f'(x)+\frac{h^2}{2} f''(x)-\frac{h^3}{6} f'''(\xi_1), \xi_1 \in (x-h,x) \\ f(x-2h)=f(x)-2hf'(x)+2h^2 f''(x)-\frac{4}{3} h^3 f'''(\xi_2), \xi_2 \in (x-2h,x)$$

So $\delta_{h,-,2} f(x)=-f'(x)+\frac{2}{3} h^2 f'''(\xi_2)-\frac{1}{3} h^3 f'''(\xi_1)$.

But how can we say something about $| \delta_{h,-,2} f(x)- f'(x)|$ now that we have $\delta_{h,-,2} f(x)=-f'(x)+ \dots$ instead of $\delta_{h,-,2} f(x)=f'(x)+ \dots$ ? (Thinking)
 
Last edited:
Mathematics news on Phys.org
  • #2


Hello! Based on the formula provided, we can see that $\delta_{h,-,2} f(x)$ is actually an approximation of $f'(x)$, with an error term of $h^2 ||f'''||_{\infty}$. This means that as $h$ gets smaller, the error term also decreases, and the approximation becomes closer to the true value of $f'(x)$. So even though the formula may seem to be calculating $-\delta_{h,-,2} f(x)$ instead of $\delta_{h,-,2} f(x)$, the error term ensures that it is still a valid approximation of $f'(x)$.

In other words, the negative sign in front of $f'(x)$ does not affect the accuracy of the approximation, as long as the error term is small enough. So we can still say that $|\delta_{h,-,2} f(x)- f'(x)|\leq h^2 ||f'''||_{\infty}$, regardless of the negative sign. I hope this helps clarify your doubts. Let me know if you have any other questions.
 

Related to How Can We Show the Bound for $\delta_{h,-,2} f(x)$?

1. How does the concept of "binding" apply to scientific research?

The concept of binding is a fundamental principle in scientific research that refers to the relationship between two or more variables. It is used to determine the effects of one variable on another and to establish causation or correlation between them.

2. What are some common techniques used to demonstrate binding in a scientific study?

There are several techniques used to demonstrate binding in scientific research, including statistical analysis, control groups, and double-blind studies. These methods help to control for extraneous variables and establish a clear relationship between the variables being studied.

3. How can we visually represent the bound in a scientific study?

Visual representations such as graphs, charts, and diagrams are often used to illustrate the bound in a scientific study. These visual aids can help to make complex data more accessible and highlight the relationship between variables.

4. What is the significance of showing the bound in scientific research?

Demonstrating the bound is crucial in scientific research as it allows researchers to draw conclusions and make predictions based on their findings. It also helps to validate the reliability and validity of the study's results.

5. How can we ensure that the bound is accurately represented in a scientific study?

To ensure the accuracy of the bound in a scientific study, researchers must carefully design their experiments and control for all potential confounding factors. They should also use appropriate statistical methods and clearly present their findings to avoid misinterpretation.

Similar threads

Replies
3
Views
1K
Replies
4
Views
809
Replies
6
Views
2K
Replies
1
Views
809
  • General Math
Replies
2
Views
1K
Replies
2
Views
2K
  • General Math
Replies
8
Views
2K
Replies
4
Views
1K
  • General Math
Replies
33
Views
2K
  • Special and General Relativity
Replies
1
Views
439
Back
Top